financial markets mcq

Réussis tes devoirs et examens dès maintenant avec Quizwiz!

You have been given this probability distribution for the holding-period return for KMP stock: What is the expected holding-period return for KMP stock? A. 10.40% B. 9.32% C. 11.63% D. 11.54% E. 10.88% BOOM - PR (O.3) - HPR (18%) NORMAL GROWTH - PR(0.5) - HPR (12%) RECESSION - PR(0.2) - HPR(-5%)

A

(I) The coupon rate is the rate of interest that the issuer of the bond must pay. (II) The coupon rate on old bonds fluctuates with market interest rates so they will remain attractive to investors. A) (I) is true, (II) false. B) (I) is false, (II) true. C) Both are true. D) Both are false.

A

6. Which of the following represent unsystematic risks? i) Labour strikes ii) Production problems iii) Management changes iv) Economic growth (a) i, ii and iii only (b) ii and iv only (c) i and iii only (d) i, ii and iv only

A

A call option on a stock is said to be out of the money if A. the exercise price is higher than the stock price. B. the exercise price is less than the stock price. C. the exercise price is equal to the stock price. D. the price of the put is higher than the price of the call. E. the price of the call is higher than the price of the put.

A

A repo agreement is (a) the sale of a security with a commitment to repurchase the same security as a specified price and on specified date (b) a bill of exchange drawn by a person and accepted by a bank. (c) an obligation backed by the "full faith and credit" of the government. (d) The dollar-denominated deposits at large London banks.

A

Assume that Bolton Company will pay a $2.00 dividend per share next year, an increase from the current dividend of $1.50 per share that was just paid. After that, the dividend is expected to increase at a constant rate of 5%. If you require a 12% return on the stock, the value of the stock is A. $28.57. B. $28.79. C. $30.00. D. $31.78. E. None of the options are correct.

A

Beta is the slope of the best fit line for the points with coordinates representing the ________ and the ________ for each one of several years. A) market rate of return; security's rate of return B) rate of inflation; rate of return for an individual security C) risk level of a stock; market rate of return D) rate of return; level of risk for an individual security

A

If a bond has a modified duration of 4.06, what would be the change in the price of bond when the interest rate changed from 150 basis points to160 basis point? (a) Bond price would decrease by 0.406%. (b) Bond price would increase by 0.406%. (c) Bond price would decrease by 4.06%. (d) Bond price would increase by 4.06%.

A

In its simplest form, a credit default swap provides A) insurance against default in the principle and interest payments of a credit instrument. B) bond investors with a method to swap interest payments for principle payments during a "credit event." C) an alternative method for bond issuers to pay principle and interest payments via a swap. D) the government with a guarantee that certain bond issues will not run into credit problems.

A

In the futures markets, gains and losses in a contract's value are calculated every day and added to or subtracted from the trader's account. This procedure is called A) mark-to-the-market. B) checking the maintenance deposit. C) checking the maintenance margin. D) settling.

A

Markets in which funds are transferred from those who have excess funds available to those who have a shortage of available funds are called A) financial markets. C) commodity markets. B) funds markets. D) derivative exchange markets.

A

New issues of securities are sold in the ________ market(s). A. primary B. secondary C. over-the-counter D. primary and secondary

A

The capital allocation line can be described as the A. investment opportunity set formed with a risky asset and a risk-free asset. B. investment opportunity set formed with two risky assets. C. line on which lie all portfolios that offer the same utility to a particular investor. D. line on which lie all portfolios with the same expected rate of return and different standard deviations.

A

The liquidity premium theory can explain (a) positively sloped yield curve (b) negatively sloped yield curve (c) flat yield curve (d) all of the above

A

The risk that the rate of return on an investment will be less than expected due to factors that are 6) independent of the investment, such as political, social or economic events, is called A) market risk. B) business risk. C) liquidity risk. D) financial risk.

A

The value of a derivative security A. depends on the value of the related security. B. is unable to be calculated. C. is unrelated to the value of the related security. D. has been enhanced due to the recent misuse and negative publicity regarding these instruments.

A

Which of the following are true concerning institutional investors? I. Institutional investors are professionals who manage money for other people. II. Banks, insurance companies and mutual funds are all institutional investors. III. Institutionalinvestorsareindividualswhoinvestindirectlythroughfinancialinstitutions. IV. Institutional investors invest large sums of money. A) I, II and IV only B) II, III and IV only C) I and II only D) I,II,IIIandIV

A

Which of the following is NOT an investment as defined in the text? A) a new automobile B) a certificate of deposit issued by a bank C) a United States Saving Bond D) a mutual fund held in a retirement account

A

Which of the following is the largest borrower in the money markets? A) The U.S. Treasury C) U.S. firms engaged in foreign trade B) Commercial banks D) Large corporations

A

Which of the following variables are part of the Black-Scholes option pricing model? I. the market price of the underlying stock II. the volatility of the underlying security III. thestrikepriceoftheoption IV. the risk-free rate of interest V. the beta of the underlying security VI. the time remaining before the option expires A) I, II, III, IV and VI only C) I, II, III, IV, V and VI B) I, II, IV and VI only D) I, II and III only

A

Which one of the following is not used to deal with systemic risk? (a) Agency capture (b) Basel Accords III (c) Centralized clearing (d) Ban on trading in `naked' credit default swap

A

Amy purchased stock A which has the following information State Probability Return Boom 0.4 30% Bust 0.6 -10% Calculate the expected return for stock A. (a) 6% (b) 10% (c) 20% (d) 18%

A (0.4 X 0.3) +06(0.1)

A bond with a market price of 110.5 at issuance (a) Is issued at par. (b) Is issued at premium. (c) Is issued at discount. (d) has a coupon rate that lower that the market rate.

B

According to the mean-variance criterion, which of the statements below is correct? (i) A, E(r) 10%, S.D. 5% (i) B, E(r) 21%, S.D. 11% (i) C, E(r) 18%, S.D. 23% (i) D, E(r) 24%, S.D. 16% A. Investment B dominates investment A. B. Investment B dominates investment C. C. Investment D dominates all of the other investments. D. Investment D dominates only investment B. E. Investment C dominates investment A.

B

In the case of an insurance policy, _______ occurs when the existence of insurance encourages the insured party to take risks that increase the likelihood of an insurance payoff; ________ occurs when those most likely to get large insurance payoffs are the ones who want to purchase insurance the most. (a) Moral hazard; insurance market discrimination (b) Moral hazard; adverse selection (c) Adverse selection; morale hazard (d) Morale hazard, insurance segregation

B

Rachel has just purchased a heating oil contract at $2.05 per gallon. The contract size is 21,000 gallons. Initial margin is $6,075. Maintenance margin is $4,500. If the price of heating oil is $2.15 when the contract expires, what is Rachel's profit or loss? (a) $2100 loss (b) $2100 profit (c) $3975 loss (d) $2400 loss

B

Structure of interest rates is A. the relationship between the rates of interest on all securities. B. the relationship between the interest rate on a security and its time to maturity. C. the relationship between the yield on a bond and its default rate. D. All of the options are correct. E. None of the options are correct.

B

The yield to maturity on a bond is A. below the coupon rate when the bond sells at a discount and equal to the coupon rate when the bond sells at a premium. B. the discount rate that will set the present value of the payments equal to the bond price. C. based on the assumption that any payments received are reinvested at the coupon rate. D. None of the options are correct.

B

When bad drivers line up to purchase collision insurance, automobile insurers are subject to the A) assigned risk problem. B) adverse selection problem. C) moral hazard problem. D) ill queue problem.

B

Which of the following are traded in the securities market? A) bonds B) stocks C) derivatives D) all of the above

B

Which one of the following statements regarding the modern portfolio theory is incorrect? (a) The effectiveness of diversification depends on the correlation or covariance between returns on the individual assets combined into portfolio. (b) Risk-averse investors would aim to hold portfolios that are efficient in that they provide the highest expected return for lowest risk. (c) Systematic risk cannot be eliminated by diversification. (d) The systematic risk is usually measured by the beta of an asset.

B

2. Which one of the following statements is incorrect? (a) Money markets exist to facilitate the exchange of securities such as treasury bills or other loans with a short time to maturity. (b) The primary market involves the issue of new shares of corporations. (c) Direct finance occurs when savers deposit funds into banks, and banks then loan these to borrowers. (d) The motive for the short-sale activities is that the asset will be purchased for a lower price and returned to its lender.

C

A credit default swap, or CDS, is essentially A) a method for swapping credit agreements between banks. B) insurance against the default of a party in a swap agreement. C) insurance against default on a financial instrument. D) a method for companies in default to swap credit ratings.

C

According to the expectation hypothesis, an upward-sloping yield curve implies that A. interest rates are expected to remain stable in the future. B. interest rates are expected to decline in the future. C. interest rates are expected to increase in the future. D. interest rates are expected to decline first, then increase. E. interest rates are expected to increase first, then decrease.

C

An inverted yield curve A) means that long-term bonds are yielding more than short-term bonds. B) rewards long-term investors for the additional risk they are assuming. C) generally results from actions by the Federal Reserve to control inflation. D) exists when intermediate-term bonds have higher yields than either short-term or long-term bonds.

C

Combining uncorrelated assets will A) cause the other assets in the portfolio to become positively related. B) not change the overall risk level of a portfolio. C) decrease the overall risk level of a portfolio. D) increase the overall risk level of a portfolio.

C

If you believe in the _______ form of the EMH, you believe that stock prices only reflect all information that can be derived by examining market trading data, such as the history of past stock prices, trading volume or short interest. A. semi-strong B. strong C. weak D. All of the options are correct. E. None of the options are correct.

C

If you buy an option to buy Treasury futures at 115, and at expiration the market price is 110, A) the call will be exercised. C) the call will not be exercised. B) the put will be exercised. D) the put will not be exercised.

C

In a well-diversified portfolio, A. market risk is negligible. B. systematic risk is negligible. C. unsystematic risk is negligible. D. non diversifiable risk is negligible.

C

Melissa owns the following portfolio of stocks. What is the return on her portfolio? Stock A B C Amount Invested $8,000 (a) $4,000 (b) $12,000(c) Return on Stock 17.5%(a) 11.0%(b) 4.3%(c) A) 9.0% B) 10.9% C) 9.8% D) 8.0%

C

Preferred stockholders hold a claim on assets that has priority over the claims of A) bondholders, but after that of common stockholders. B) both common stockholders and bondholders. C) common stockholders, but after that of bondholders. D) neither common stockholders nor bondholders.

C

Rachel, an adventurous investor, bought a futures contract for May delivery of 40,000 lbs of lean 20) pork. In late April, she is becoming uneasy about what to do with so much pork. A) She can try to find someone willing to take delivery of the pork. B) She needs to buy a very large freezer. C) She can sell the contract or cancel it by entering into the opposite contract (to sell rather than buy). D) She can choose settle her profit or loss in cash rather than pork.

C

Sarah purchased a stock one year ago at a price of $32 a share. In the past year, she has received 4) four quarterly dividends of $0.75 each. Today she sold the stock for $38 a share. Her capital gain per share is A) $(6.00). B) $9.00. C) $6.00. D) $3.00.

C

The duration of a 5-year zero-coupon bond is A. smaller than 5. B. larger than 5. C. equal to 5. D. equal to that of a 5-year 10% coupon bond. E. None of the options are correct.

C

The risk-free rate of return is 2.2 percent, the expected market return is 11 percent, and the beta for Solstice, Inc. is 1.12. What is Solstice's required rate of return? A) 13.20% B) 8.80% C) 12.05% D) 14.30%

C

The standard deviation of a portfolio of risky securities is A. the square root of the weighted sum of the securities' variances. B. the square root of the sum of the securities' variances. C. the square root of the weighted sum of the securities' variances and covariances. D. the square root of the sum of the securities' covariances.

C

The stock of a technology company has an expected return of 15% and a standard deviation of 20% 8) The stock of a pharmaceutical company has an expected return of 13% and a standard deviation of 18%. A portfolio consisting of 50% invested in each stock will have an expected return of 14 % and a standard deviation A) the average of 20% and 18%. B) the answer cannot be determined with the information given. C) less than the average of 20% and 18%. D) greater than the average of 20% and 18%.

C

The tendency of investors to take greater risks after a large loss and fewer risks after a large gain can be attributed to (a) overconfidence (b) framing (c) loss aversion (d) representativeness

C

The usual maturity range for commercial paper is ________. A) 4, 13, and 26 weeks B) 1 to 7 days C) 1 to 270 days D) 1 to 15 days

C

The writer of a put A) accepts the obligation to sell a predetermined number of shares at a predetermined price. B) is betting the price of the underlying security will increase in value. C) is hoping that the put will be in-the-money prior to expiration. D) will pay the premium whether or not the option is exercised.

C

Which of the following are needed to determine the appropriate value of a bond? I. required rate of return II. time to maturity III. frequency of interest payments IV. coupon rate A) II and III only C) I, II, III and IV B) II, III and IV only D) III and IV only

C

Which one of the following has the lowest level of risk? A) money market mutual fund account B) banker's acceptance C) U.S. Treasury bill D) commercial paper

C

Which one of the following is not true of derivative markets? (a) Derivative markets are used to hedge risk. (b) Derivative markets are used to make a profit on price anomalies on financial instruments (c) Derivative markets are financial markets where traders buy and sell shares for immediate delivery. (d) Speculators use derivative markets to obtain profit from price changes.

C

Which one of the following money market instruments has the lowest default risk? (a) Commercial papers (b) Certificates of deposits (c) Treasury bills (d) Bankers' acceptance

C

Which one of the following statements is incorrect for relative valuation? (a) Easy to compare with competitors (b) Easy to compare with industry average (c) Normally used for firms pay dividends (d) Normally used for privately held firms

C

Winifred, Inc. paid $1.64 as an annual dividend per share last year. The company is expected to increase their annual dividends by 3% each year. How much should you pay to purchase one share of this stock if you require a 9% rate of return on this investment? A) $27.33 B) $18.22 C) $28.15 D) $18.77

C

A stock currently sells for $25 per share and pays $0.24 per year in dividends. What is an investor's valuation of this stock if she expects it to be selling for $30 in one year and requires a 15 percent return on equity investments? A) $27.74 B) $26.09 C) $30.24 D) $26.30

D

Compared to money market securities, capital market securities have A) less risk. C) lower yields. B) more liquidity. D) longer maturities.

D

Financial intermediaries A) are involved in the process of indirect finance. B) exist because there are substantial information and transaction costs in the economy. C) improve the lot of the small saver. D) do all of the above. E) do only A and B of the above.

D

Followers of the random walk hypothesis believe that A) support levels and resistance lines, when combined with basic chart formations, yield both buy and sell signals. B) that traders can earn higher than normal returns by exploiting market anomalies such as the small-firm effect. C) security analysis is the best tool to utilize when investing in the stock market. D) the price movements of stocks are unpredictable, and therefore security analysis will not help to predict future market behavior.

D

If a portfolio manager believes stock prices will fall and knows that a block of funds will be received in the future, then he should A) stay out of the futures market. B) borrow and buy securities now. C) buy stock index futures long. D) sell stock index futures short.

D

Other things equal, diversification is most effective when A. securities' returns are uncorrelated. B. securities' returns are positively correlated. C. securities' returns are high. D. securities' returns are negatively correlated.

D

The advantage of forward contracts over futures contracts is that forward contracts A) are more liquid. B) are standardized. C) have lower default risk. D) are none of the above.

D

Which of the following represent unsystematic risks? I. the president of a company suddenly resigns II. the economy goes into a recessionary period III. acompany'sproductisrecalledfordefects IV. the Federal Reserve unexpectedly changes interest rates A) I, II and III only B) II and IV only C) I, II and IV only D) I and III only

D

Which of the following securities is a money market instrument? A. Treasury note B. Treasury bond C. Municipal bond D. Commercial paper

D

Which of the following statements about the characteristics of debt and equity are true? A) They both involve a claim on the issuer's income. B) They both enable a corporation to raise funds. C) They both can be long-term financial instruments. D) All of the above. E) Only A and B of the above.

D

Which one of the following statements regarding financial institutions is incorrect? (a) Mutual funds pool the resources of many small investors by selling these investors shares and using the proceeds to buy securities. (b) The process of underwriting a stock or bond issue requires that the investment bank purchases the entire issue at a predetermined price and then resell it in the market. (c) A venture capital firm is a specialized firm that finances young, start-up companies. (d) Hedge funds only focus on hedging.

D

Which one of the following statements regarding the Capital Market Line (CML) is incorrect? (a) The CML shows the trade-off between expected return and risk for all efficient portfolios. (b) The CML is an efficient set of all portfolios that provides the investor with the best possible investment opportunities when a risk-free asset is available. (c) The slope of the CML measures the market price of risk. (d) According the CML, the best portfolio is the market portfolio regardless of investors' utility.

D

With regard to a futures contract, the long position is held by A. the trader who bought the contract at the largest discount. B. the trader who has to travel the farthest distance to deliver the commodity. C. the trader who plans to hold the contract open for the lengthiest time period. D. the trader who commits to purchasing the commodity on the delivery date. E. the trader who commits to delivering the commodity on the delivery date.

D

You purchase a share of Boeing stock for $90. One year later, after receiving a dividend of $3, you sell the stock for $92. What was your holding-period return? A. 4.44% B. 2.22% C. 3.33% D. 5.56% E. None of the options are correct.

D

________ are an example of a financial institution. A) Insurance companies C) Banks B) Finance companies D) All of the above

D

________ bias means that investors are too slow in updating their beliefs in response to evidence. A. Framing B. Regret avoidance C. Overconfidence D. Conservatism E. None of the options are correct.

D

________ enables mutual funds to consistently outperform a randomly selected group of stocks A) Diversification C) Managerial expertise B) Denomination intermediation D) None of the above

D

Jane bought 100 shares of Erain Plc. She sold the shares after two years at the price of 20. Over the two years, the company paid dividends of £1 and £1.5 per share each year. The interest rate remained the same for two years at 10%. What would be the current fair price for the stock? (a) £20.8 (b) £19.6 (c) £22.5 (d) £18.68

D (1/1.1)+(20+1.5)/(1.1^2)=18.68

Stock XYZ had the following prices over the last three days since the company was listed, 10, 15, 12. What is the standard deviation of the stock returns? Assuming no dividends paid. (a) 12.25% (b) 35.00% (c) 15% (d) 49%

D R1=15/10-1=50% R2=12/15-1=-20% Average Return=(50%-20%)/2=15% SD=[((0.5-0.15)^2+(-0.2-0.15)^2)/(2-1)]^0.5=49%

According to the Gordon growth model, what is an investor's valuation of a stock whose current dividend is $1.00 per year if dividends are expected to grow at a constant rate of 10 percent over a long period of time and the investor's required return is 15 percent? A) $20 B) $7.33 C) $11 D) $4.40 E) $22

E

According to the efficient market hypothesis A) information in newspapers and in the published reports of financial analysts is already reflected in market prices. B) one cannot expect to earn an abnormally high return by purchasing a security. C) unexploited profit opportunities abound, thereby explaining why so many people get rich by trading securities. D) all of the above are true. E) only A and B of the above are true.

E

Important implications of the efficient market hypothesis include which of the following? A) Future changes in stock prices should, for all practical purposes, be unpredictable. B) Stock prices will respond to announcements only when the information in these announcements is new. C) Sometimes a stock price declines when good news is announced. D) All of the above. E) Only A and B of the above.

E

Proponents of the EMH typically advocate A. an active trading strategy. B. investing in an index fund. C. a passive investment strategy. D. an active trading strategy and investing in an index fund. E. investing in an index fund and a passive investment strategy.

E

Which one of the following is the function of financial markets? (a) Clearing and settling payments (b) Transferring resources across time and space (c) Managing risk (d) Both b and c (e) All of the above

E


Ensembles d'études connexes

Online quiz questions from Chapts 6, 7, 8, 9, 11, 14, 15

View Set

can exercise change your brain 253 lecture 16

View Set

Theories and Concepts: Pain and Comfort

View Set

Human Anatomy Chapter Three: Practice Questions

View Set

The Scientific Revolution (1543 - 1687)

View Set

Physics - U2 L13 - Correlation and Causation

View Set